Inscription / Connexion Nouveau Sujet

1 2 3 +


Niveau Maths sup
Partager :

Inequation

Posté par
jb2017
14-07-17 à 10:39

Bonjour
Je n'arrive pas à résoudre ce problème merci d'avance pour la solution:

Soit  a,b,c>0 vérifiant a^3+b^3+c^3=3. 
 \\
Déterminer \alpha=\min (\dfrac{a^3}{a+b}+\dfrac{c^3}{a+c}+\dfrac{b^3}{b+c})

\alpha=\dfrac{3}{2} ?

Posté par
Glapion Moderateur
re : Inequation 14-07-17 à 12:47

oui, on se doute vu la symétrie du problème que le minimum est pour a=b=c ce qui donne 3/2 pour .

Reste à démontrer proprement que \dfrac{a^3}{a+b}+\dfrac{c^3}{a+c}+\dfrac{b^3}{b+c}  \geq \dfrac{3}{2} si a^3+b^3+c^3=3

Essaye Cauchy-Schwartz avec x1=(a^3/(a+b))1/2 ; x2 = (b^3/(b+c))1/2 ; x3 = (c^3/(c+a))1/2 et y1=(a^3(a+b))1/2 ; y2 = etc ....

rappel CS c'est ( xiyi(xi²)(yi²)

Posté par
Razes
re : Inequation 14-07-17 à 13:30

Bonjour,

Tu pourrais appliquer le Multiplicateur de Lagrange   pour minimiser la fonction f

f(a,b,c,d)=\dfrac{a^3}{a+b}+\dfrac{c^3}{a+c}+\dfrac{b^3}{b+c}

g(a,b,c,d)=a^3+b^3+c^3-3

Posté par
Razes
re : Inequation 14-07-17 à 13:32

Correction:

f(a,b,c)=\dfrac{a^3}{a+b}+\dfrac{c^3}{a+c}+\dfrac{b^3}{b+c}

g(a,b,c)=a^3+b^3+c^3-3

Posté par
Razes
re : Inequation 14-07-17 à 14:49

\exists a^*, b^*,c^*; \exists \lambda ^*\mid \triangledown f(a^*, b^*,c^*)+\lambda ^*\triangledown g(a^*, b^*,c^*)=0;g(a^*, b^*,c^*)=0


\triangledown f(a, b,c)=\begin{pmatrix} \dfrac{3a^2\left ( a+b \right )-a^3}{\left ( a+b \right )^2}\\ \dfrac{3b^2\left ( b+c \right )-b^3}{\left ( b+c\right )^2}\\ \dfrac{3c^2\left ( c+a \right )-c^3}{\left ( c+a\right )^2} \end{pmatrix}

\triangledown g(a, b,c)=\begin{pmatrix} 3a^2\\3b^2\\3c^2\\ \end{pmatrix}

Déterminer \lambda ^*, puis  f(a^*, b^*,c^*)

Posté par
jb2017
re : Inequation 14-07-17 à 15:29

Merci pour vos réponses.
@Razes, évidemment j'ai regardé la méthode des  multiplicateurs de Lagrange mais cela ne marche pas très souvent car  les calculs sont souvent impossibles comme ici, sauf erreur de ma part.
En appliquant  l'idée de Glapion on remplace le problème par une majoration
à effectuer sur la surface (S) d'équation a^3+b^3+c^3=3, a,b,c>0.
Plus précisément  il faut montrer que
g(a,b,c)=a^3 (a+b)+c^3 (a+c)+b^3 (b+c)\leq 6
Cela a au moins le mérite de remplacer les quotient par des produits et une minoration par une majoration. Donc je préfère cette nouvelle formulation
Néanmoins je me retrouve avec des difficultés un peu analogues que j'ai déjà rencontrées sauf que l'on a des produits maintenant.

L'idée que j'ai suivie, c'est de considérer un vecteur u tangent à (S), donc orthogonal à
(a^2,b^2,c^2) et de me placer en un point x_0=(a,b,c)\neq(1,1,1). En montrant
que la dérivée de g(x_0+h u)  par rapport à h, en h=0 n'est pas nulle (pour au moins un vecteur u bien choisi) alors il n'y a pas d'extremum et le résultat est acquis.
Bien que les expressions soient maintenant polynomiales cela me semble difficile.





javascript:format('tex')

Posté par
Razes
re : Inequation 14-07-17 à 15:40

Les qutients vont disparaitre.

As tu au moins éssayé?

Posté par
jb2017
re : Inequation 14-07-17 à 16:49

@Rasez
Je ne comprends ta remarque?
Je donne d'ailleurs l'expression, g(a,b,c),   elle n'est pas tombée du ciel.

Posté par
DOMOREA
re : Inequation 14-07-17 à 16:55

bonjour,
Notons m=min(a,b,c), m\le 1
alors \frac{a^3}{a+b}+\frac{c^3}{a+c}+\frac{b^3}{b+c}\ge \frac{3}{2m}
or  \frac{3}{2m}\ge \frac{3}{2}
il est évident que la valeur \frac{3}{2} est obtenue avec a=b=c=1
\frac{3}{2}est le minimum

Posté par
DOMOREA
re : Inequation 14-07-17 à 17:10

re ,
je pense que j'ai écris une énormité, j'avais travaillé avec le maximum, puis j'ai changé de braquet et mes pédales se sont mélangées
tout est donc à revoir

Posté par
larrech
re : Inequation 14-07-17 à 17:56

Bonjour Razes

Ne reste-t-il pas un  \Large-\frac{c^3}{(a+c)^2} dans l'expression de \Large\frac{\partial f}{\partial a} ? Et des termes analogues dans les autres.

Posté par
Razes
re : Inequation 14-07-17 à 18:19

larrech @ 14-07-2017 à 17:56

Bonjour Razes

Ne reste-t-il pas un  \Large-\frac{c^3}{(a+c)^2} dans l'expression de \Large\frac{\partial f}{\partial a} ? Et des termes analogues dans les autres.

Effectivement, il manque un terme dans chaque ligne.

Posté par
jb2017
re : Inequation 14-07-17 à 21:57

Bonsoir
merci pour ceux qui ont essayé mais je crois que ce n'est pas donné.

Posté par
Alexique
re : Inequation 14-07-17 à 23:07

Bonsoir,

application 3 page 11 en remplaçant les numérateurs par des cubes : C'est souvent plus élégant à résoudre sans analyse mais demande un peu plus d'originalité et d'efforts...

Posté par
Razes
re : Inequation 15-07-17 à 00:23

Alexique @ 14-07-2017 à 23:07

Bonsoir,

application 3 page 11 en remplaçant les numérateurs par des cubes : C'est souvent plus élégant à résoudre sans analyse mais demande un peu plus d'originalité et d'efforts...

Merci pour le document, c'est très intéressant.

Avec l'inégalité de Chebyshev, la résolution est plus élégante.

Posté par
jb2017
re : Inequation 15-07-17 à 00:54

Bonsoir
Merci @alexique, évidemment  on peut (ou on devrait?) espérer une solution sans analyse, c'est à dire avec des moyens élémentaires. C'est même un peu pour cela que j'ai posé la question ici. Mais je ne vois pas le rapport de ma question avec la référence  que tu donnes. D'une part j'ai a minimiser une fonctionnelle sous contrainte (il n' y en n'a pas
dans ta référence). Et puis les numérateurs sont des cubes.  
@Razes. Détaille un peu ta solution je ne la comprend pas.
Dans les conditions actuelles, pour moi ce problème reste ouvert.  

Posté par
jb2017
re : Inequation 15-07-17 à 00:55

@Razes.  excuses moi mais j''ai oublié de mettre s-t-p!!

Posté par
Alexique
re : Inequation 15-07-17 à 15:48

@jb : ma référence n'utilise pas d'analyse, c'est justement pour cela que la résolution est plus élégante et moins calculatoire. Par contre, je concède que ton problème n'est pas symétrique en les variables ce qui m'embête pour appliquer Tchebychev... Il faudrait creuser un peu ou poster sur mathématiques.net..

Posté par
lake
re : Inequation 15-07-17 à 18:55

Bonjour,

Citation :
Il faudrait  poster sur mathématiques.net..


C' est fait:

Mais ça coince aussi là bas...

Posté par
Razes
re : Inequation 16-07-17 à 04:02

Bonsoir,

La méthode du Multiplicateur de Lagrange me paraissait une bonne solution (surtout que je l'avait déjà utilisé) mais avec les dénominateurs les calculs se compliquent.

Utilisons l'inégalité de Cauchy-Schwarz :

\left|\sum _{{i=1}}^{n}x_{{i}}y_{{i}}\right|\leqslant \left(\sum _{{i=1}}^{n}x_{{i}}^{{2}}\right)^{{1/2}}\left(\sum _{{i=1}}^{n}y_{{i}}^{{2}}\right)^{{1/2}}.

x_1=\sqrt{a+b},x_2=\sqrt{b+c},x_3=\sqrt{c+a},y_1=\dfrac{1}{\sqrt{a+b}},y_2=\dfrac{1}{\sqrt{b+c}},y_3=\dfrac{1}{\sqrt{c+a}}
Donc:

\left ( \sqrt{a+b}^2 +\sqrt{b+c}^2+\sqrt{c+a}^2\right )\left ( \dfrac{1}{\sqrt{a+b}^2}+ \dfrac{1}{\sqrt{b+c}^2}+\dfrac{1}{\sqrt{c+a}^2}\right )\geqslant 3^2\Leftrightarrow 
 \\ 
 \\ 2(a+b+c)\left (\dfrac{1}{a+b}+\dfrac{1}{b+c}+\dfrac{1}{c+a}\right )\geqslant 9\Leftrightarrow \left (\dfrac{1}{a+b}+\dfrac{1}{b+c}+\dfrac{1}{c+a}\right )\geqslant \dfrac{9}{2(a+b+c)} (2)

a, b, c\geqslant 0
De plus a^3+b^3+c^3=1; donc a, b, c\leqslant 1\Rightarrow a+b+c\leqslant 3\Rightarrow  \dfrac{1}{a+b+c}\geqslant \dfrac{1}{3}(3)

L'expression (2) devient donc: \left (\dfrac{1}{a+b}+\dfrac{1}{b+c}+\dfrac{1}{c+a}\right )\geqslant \dfrac{3}{2}(4)

Utilisons Inégalité de Tchebychev pour les sommes :

On peut facilement ordonner a,b,c pour remplir les conditions nécessaires. (c'est facile)

{\displaystyle {1 \over n}\sum _{k=1}^{n}a_{k}b_{k}\geq \left({1 \over n}\sum _{k=1}^{n}a_{k}\right)\left({1 \over n}\sum _{k=1}^{n}b_{k}\right).}

\frac{1}{3}\left (\dfrac{a}{a+b}+\dfrac{b^3}{b+c}+\dfrac{c^3}{c+a}\right )\geqslant \frac{1}{3}\left ( a^3+b^3+c^3 \right )\frac{1}{3}\left (\dfrac{1}{a+b}+\dfrac{1}{b+c}+\dfrac{1}{c+a}\right )\Rightarrow 
 \\ 
 \\ \left (\dfrac{a}{a+b}+\dfrac{b^3}{b+c}+\dfrac{c^3}{c+a}\right )\geqslant \left (\dfrac{1}{a+b}+\dfrac{1}{b+c}+\dfrac{1}{c+a}\right ) (5)


(4) et (5) nous permettent de conclure que: \left (\dfrac{a}{a+b}+\dfrac{b^3}{b+c}+\dfrac{c^3}{c+a}\right )\geqslant\dfrac{3}{2}

Posté par
Razes
re : Inequation 16-07-17 à 04:08

J'avais oublié l'exposant de a^3

Utilisons Inégalité de Tchebychev pour les sommes :

On peut facilement ordonner a,b,c pour remplir les conditions nécessaires. (c'est facile)

{\displaystyle {1 \over n}\sum _{k=1}^{n}a_{k}b_{k}\geq \left({1 \over n}\sum _{k=1}^{n}a_{k}\right)\left({1 \over n}\sum _{k=1}^{n}b_{k}\right).}

\frac{1}{3}\left (\dfrac{a^3}{a+b}+\dfrac{b^3}{b+c}+\dfrac{c^3}{c+a}\right )\geqslant \frac{1}{3}\left ( a^3+b^3+c^3 \right )\frac{1}{3}\left (\dfrac{1}{a+b}+\dfrac{1}{b+c}+\dfrac{1}{c+a}\right )\Rightarrow \\ \\ \left (\dfrac{a^3}{a+b}+\dfrac{b^3}{b+c}+\dfrac{c^3}{c+a}\right )\geqslant \left (\dfrac{1}{a+b}+\dfrac{1}{b+c}+\dfrac{1}{c+a}\right )    (5)


(4) et (5) nous permettent de conclure que: \left (\dfrac{a^3}{a+b}+\dfrac{b^3}{b+c}+\dfrac{c^3}{c+a}\right )\geqslant\dfrac{3}{2}

Posté par
DOMOREA
re : Inequation 16-07-17 à 09:06

bonjour Razes,
Tu écris à un moment  a+b+c\le 3 c'est sans doute vrai mais pour cela tu utilises a,b,c \le 1 ce qui est faux. Comment démontrer que  a+b+c\le 3
Pour l'utilisation de l'inégalité de Tchebychev , tu es donc il me semble dans le cas:
a^3\ge b^3 \ge c^3 et \frac{1}{a+b}\ge\frac{1}{b+c}\ge \frac{1}{c+a}
il y a un problème car si b\ge c alors a+b \ge c+a et donc \frac{1}{a+b}\le \frac{1}{c+a}

Posté par
Razes
re : Inequation 16-07-17 à 13:45

a,b,c\geqslant 0; d'après l'énoncé

a^3+b^3+c^3=3
Inégalité de Hölder
 \sum _{{k=1}}^{n}|x_{k}\ y_{k}|\leq \left(\sum _{{k=1}}^{n}|x_{k}|^{p}\right)^{{1/p}}\left(\sum _{{k=1}}^{n}|y_{k}|^{q}\right)^{{1/q}}.

\left (\dfrac{a}{a+b+c}+\dfrac{b}{a+b+c}+\dfrac{c}{a+b+c}\right )\leqslant \left ( a^3+b^3+c^3 \right )^\frac{1}{3}\left ( \left (\dfrac{1}{a+b+c}\right )^\frac{3}{2}+\left (\dfrac{1}{a+b+c}\right )^\frac{3}{2}+\left (\dfrac{1}{a+b+c}\right )^\frac{3}{2} \right )^\frac{2}{3}\Leftrightarrow 
 \\ 1\leqslant 3^\frac{1}{3}3^\frac{2}{3}\dfrac{1}{a+b+c}\Leftrightarrow

\frac{1}{3}\leqslant\dfrac{1}{a+b+c}
CQFD

Posté par
Razes
re : Inequation 16-07-17 à 14:39

DOMOREA @ 16-07-2017 à 09:06

bonjour Razes,
Tu écris à un moment  a+b+c\le 3 c'est sans doute vrai mais pour cela tu utilises a,b,c \le 1 ce qui est faux. Comment démontrer que  a+b+c\le 3
Pour l'utilisation de l'inégalité de Tchebychev , tu es donc il me semble dans le cas:
a^3\ge b^3 \ge c^3 et \frac{1}{a+b}\ge\frac{1}{b+c}\ge \frac{1}{c+a}
il y a un problème car si b\ge c alors a+b \ge c+a et donc \frac{1}{a+b}\le \frac{1}{c+a}

Bonjour DOMOREA,
J'ai répondu à ta première question dans mon précédent message. C'était une erreur d'inattention. Bien que l'expression a+b+c=<3 me semblait évidente. Bien qu'il faut tout justifier.

Pour la seconde partie, l'inégalité de Schebychev est valable pour les deux cas. Voir le lien wiki.

Posté par
DOMOREA
re : Inequation 16-07-17 à 15:27

oui, j'ai bien vu les deux cas, mais alors as-tu l'inégalité qui convient?
a^3\ge b^3\ge c^3 et \frac{1}{a+b}\le\frac{1}{a+c}\le\frac{1}{c+b}
avec ceci l'inégalité est dans l'autre sens.
excuse moi si je dis des bétises

Posté par
DOMOREA
re : Inequation 16-07-17 à 16:14

re
Bonjour tout le monde,
Après réflexion, il me semble qu'il y a une erreur dans le texte.
Il faudrait lire :
\frac{a}{b+c}+\frac{b}{c+a}+\frac{c}{a+b} et c'est alors une fonction symétrique des 3 variables a,b,c  et dans ce cas la difficulté que j'ai soulevée pour Razes disparaît.
Avec cette fonction, en plus,  la méthode du multiplicateur de Lagrange sur laquelle j'ai bataillé donnerait  des vecteurs bien plus simples.
Qu'en pensez-vous ?

Posté par
DOMOREA
re : Inequation 16-07-17 à 16:16

errata, a^3 ,b^3,c^3 et non pas a,b,c au numérateur

Posté par
jb2017
re : Inequation 16-07-17 à 17:00

Bonjour
merci @Domorea pour suivre la proposition de @Razes qui a de bonnes idées. Effectivement il n'a pas de problème pour montrer que a+b+c\leq 3 mais on n'a pas les hypothèses pour appliquer l'inégalité de Tchebychev.
Maintenant il n'y a pas d'erreur dans l'énoncé. La difficulté vient du fait que l'expression
n'est pas symétrique en les variables a,b,c. On a le droit de changer l'énoncé mais c'est un autre exercice qui est certainement plus facile.
D'autre part j'ai vérifié numériquement l'inégalité proposée, elle est correcte et le triplet (1,1,1) est bien l'unique point où le minimum (ou bien le maximum pour le problème dual) est atteint.
Sur le forum  proposé par @lake et@alexique ce problème a été posé et la solution n'a pas été trouvée. Sur un forum  étranger  un peu spécialisé sur les inégalités personne n'a donné de réponses.
J'ai proposé cette question sur ce site car on peut espérer une solution avec des moyens élémentaires.
Personnellement j'ai la solution ssi j'arrive à démontrer qu'un certain
polynôme  (un peu horrible)  admet comme unique solution 1 dans l'intervalle
[1 , 3^{1/3}]
Evidemment c'est facile à vérifier numériquement pour ce  polynôme.
Cependant je l'ai perdu en pensant que le problème avait une solution plus élégante (i.e moins calculatoire).

Si je ne trouve pas ici de solution, j'essaierai de refaire le calcul et le proposerait comme un problème indépendant.  




Posté par
jb2017
re : Inequation 16-07-17 à 17:08

je me suis trompé le problème est encore plus compliqué que ce que j'ai dit.
Le polynôme est à 2 variables et il faut montrer que  l'unique solution dans le carré [0,3^{1/3}]^2 est le  couple (1,1).

Posté par
Razes
re : Inequation 16-07-17 à 17:22

DOMOREA @ 16-07-2017 à 16:14

re
Bonjour tout le monde,
Après réflexion, il me semble qu'il y a une erreur dans le texte.
Il faudrait lire :
\frac{a}{b+c}+\frac{b}{c+a}+\frac{c}{a+b} et c'est alors une fonction symétrique des 3 variables a,b,c  et dans ce cas la difficulté que j'ai soulevée pour Razes disparaît.
Avec cette fonction, en plus,  la méthode du multiplicateur de Lagrange sur laquelle j'ai bataillé donnerait  des vecteurs bien plus simples.
Qu'en pensez-vous ?


Ce sont des expressions cycliques.

Posté par
carpediem
re : Inequation 17-07-17 à 22:02

salut

a, b et c sont positifs donc

s = \dfrac {a^3} {a + b} + \dfrac {b^3} {b + c} + \dfrac {c^3} {c + a} \ge \dfrac {a^3} {a + b + c} + \dfrac {b^3} {b + c + a} + \dfrac {c^3} {c + a + b} = \dfrac 3 {a + b+ c}


or d'après l'inégalité de holder : a + b + c \le (a^3 + b^3 + c^3)^{1/3} (1 + 1 + 1)^{2/3} = 3 \iff \dfrac 1 {a + b + c} \ge \dfrac 1 3

et par conséquent s \ge 1


désolé ... ça ne suffit pas ...

Posté par
DOMOREA
re : Inequation 18-07-17 à 15:38

bonjour,
peut-être une autre piste ?
f(3^{\frac{1}{3}},0,0)>\frac{3}{2}
f(0,3^{\frac{1}{3}},0)>\frac{3}{2}
f(0,0,3^{\frac{1}{3}})>\frac{3}{2}
f(1,1,1)=\frac{3}{2}
or
g: (x,y) \rightarrow g(x,y)=\frac{x^3}{x+y}=x^3\times \frac{1}{x+y} est une fonction convexe comme produit de 2 fonctions convexes positives respectivement sur \mathbb{R} et \mathbb{R}^2
f(a,b,c)=g(a,b)+g(a,c) +g(b,c) est la somme de 3 fonctions convexes positives donc est une fonction convexe sur le domaine défini par la contrainte.
avec les remarques numériques données au début, ne pourrait-on pas dire que
f(a,b,c)\ge 3/2 sur le domaine ?

Posté par
DOMOREA
re : Inequation 18-07-17 à 15:53

re
Pour les 3 affirmations du début
avec x,y,z non nuls
f(x,0,0) ainsi que f(0,y,0) et f(0,0,z) ne sont pas définis mais je  fais un prolongement par continuité

Posté par
DOMOREA
re : Inequation 18-07-17 à 16:38

re,
je corrige et je complète
Je parle de prolongement par continuité, mais c'est absurde, il faut dire plutôt que quand (b,c) tend vers (0,0) a tend vers 3^{\frac{1}{3} et donc f(a,b,c) tend vers + infinI
idem pour les 2 autres sommets du triangle. (a,b) tendant vers (0,0) et  (a,c) tendant vers 0
De plus il faut regarder la convexité des courbes aux bords, par exemple c=0 b^3+c^3=3 f(a,b,0)=\frac{a^3}{a+b}+ b^2 sur le segment ouvert ]A(a,0),B(0,b)[
il s'agit bien d'une fonction convexe, idem pour les autres bords
Ensuite il faut tenir compte de la symétrie de la surface .
Il me semble que l'on peut montrer ( facilement ?) que l'existence d'un point (a,b,c) distinct de (1,1,1) pour lequel  on aurait f(a,b,c)<3/2  conduirait à une contradiction avec la convexité

Posté par
jb2017
re : Inequation 18-07-17 à 16:38

Bonjour @DOMOREA
Pas de problème pour les prolongement aux bord de la surface.
J'essaie de comprendre ton idée. La fonction est convexe et puis en (1,1,1) on a un minimum local (c'est facile à vérifier) et donc tu en déduirai le résultat.

Dans le cas 1-dimensionnel (fonction à une variable régulière) si tu as une fonction convexe définie sur un intervalle et si x_0 est un minimum local alors c'est bien bien un minimum global car la courbe reste au dessus de chaque tangente.  
Franchement je dois réfléchir ici mais ton idée est bien à regarder de  plus près.

Posté par
DOMOREA
re : Inequation 18-07-17 à 16:46

re,
intuitivement et pour vous expliquer ma vision du problème,  je vois  comme une surface triangulaire équilatéral  à la base et dont on soulèverait les 3 sommets vers l'infini , ce qui formerait une poche dont la partie inférieure serait le centre de gravité du triangle restant à la hauteur 3/2

Posté par
jb2017
re : Inequation 18-07-17 à 17:26

Ce qui voudrait dire que si  (u,v) est un paramétrage de S
(u,v)-->h(u,v)=f(x(u,v),y(u,v),z(u,v)) est une surface convexe.
Il y a toujours moyen de calculer (théoriquement) la matrice des dérivées secondes de h en chaque (u,v) est voir si elle est définie positive
mais du point de vue calculs ce n'est pas garanti car on doit faire d'une façon générale.
Bon ici je n'ai plus le temps mais l faut voir.

Posté par
DOMOREA
re : Inequation 18-07-17 à 17:36

bonjour jb2017
merci de considérer la piste que je propose.

je continue donc:
Sur le bord (0,b,c) la fonction f peut s'écrire indifférement avec bc\neq 0    
       \frac{3+b(3-b^3)^{\frac{2}{3}}}{b+(3-b^3)^{\frac{1}{3}}} ou  \frac{3+c(3-c^3)^{\frac{2}{3}}}{c+(3-c^3)^{\frac{1}{3}}}
Par symétrie et convexité, le minimum est obtenu  pour b=c donc b=c=\frac{3^{\frac{1}{3}}}{2}  la valeur de f en ce point est supérieur à 3/2 ce qui conforte l'intuition.
Maintenant pour revenir à la question " existe-t-il un point (a0,b0,c0) différent de (1,1,1) pour lequel f serait inférieur à 3/2 ?"
On peut considérer  les points (b0,a0,c0); (a0,c0,b0),...  etc obtenus par permutation , ils auraient de même  pour image une valeur  inférieurs à 3/2 et on voit là une bonne contradiction avec la convexité de la surface.

Posté par
DOMOREA
re : Inequation 18-07-17 à 18:16

re,

Au début de ce qui précède, j'ai dit une bêtise  Pour a=0     b et c ne jouent pas le même rôle, il est nécessaire de calculer la dérivée   par rapport à x  de  :
f(x)= \frac{3+x(3-x^3)^{\frac{2}{3}}}{x+(3-x^3)^{\frac{1}{3}}}
quelle galère !

Posté par
jb2017
re : Inequation 18-07-17 à 22:19

Quelle galère!  Bien sûr tout le monde échoue et je ne sais pas si quelqu'un à la solution
quelque part .
Numériquement c'est vérifié.
Donc c'est toujours intéressant de poser un problème compréhensible par tout le monde mais néanmoins difficile.

Posté par
nadiasoeur123
re : Inequation 20-07-17 à 11:25

Bonjour ;

Comme on a : (a;b;c)\in\mathbb R^{*+3} tel que a^3+b^3+c^3=3 , procédons par disjonction de cas .

1) a; b et c sont tous inférieurs à 1 .

0<a<1 \Rightarrow 0<a^3<1
0<b<1 \Rightarrow 0<b^3<1
0<c<1 \Rightarrow 0<c^3<1

donc : 0<a^3+b^3+c^3<3

donc ce cas est à écarter .

2) a; b et c sont tous supérieurs à 1 .

 a>1 \Rightarrow a^3>1
 b>1 \Rightarrow b^3>1
 c>1 \Rightarrow c^3>1

donc :  a^3+b^3+c^3>3

donc ce cas est à écarter .

3) a = b = c = 1 .

\dfrac{a^3}{a+b}+\dfrac{b^3}{b+c}+\dfrac{c^3}{c+a} = \dfrac{3}{2} .

4) a = b = 1 et c \ne 1 .

a^3+b^3+c^3 = 2+c^3 = 3 \Rightarrow c^3 = 1 \Rightarrow c = 1 ,

donc ce cas est à écarter .

De même pour : a = c = 1 et b \ne 1 et b = c = 1 et a \ne 1

5) a = 1 et (b et c) \ne 1 .

On a : b^3 + c^3 = 2 \Rightarrow c^3=2-b^3 > 0 \Rightarrow 0<b<\sqrt[3]{2} et c = \sqrt[3]{2-b^3} , donc :

\dfrac{a^3}{a+b}+\dfrac{b^3}{b+c}+\dfrac{c^3}{c+a} = \dfrac{1}{1+b}+\dfrac{b^3}{b+\sqrt[3]{2-b^3}}+\dfrac{2-b^3}{1+\sqrt[3]{2-b^3}} .

En étudiant la fonction f définie sur ]0;1[\cup ]1;\sqrt[3]{2}[ par ; f(x) = \dfrac{1}{1+x}+\dfrac{x^3}{x+\sqrt[3]{2-x^3}}+\dfrac{2-x^3}{1+\sqrt[3]{2-x^3}} on trouve que f admet pour minimum : 1,24222017359 < \dfrac{3}{2} qui est atteint pour x = 1,23645231557 .

Conclusion :  \forall (a;b;c)\in\mathbb R^{*+3} tel que a^3+b^3+c^3=3 on a :

\dfrac{a^3}{a+b}+\dfrac{b^3}{b+c}+\dfrac{c^3}{c+a} \ge \dfrac{3}{2} .

On a : \dfrac{a^3}{a+b}+\dfrac{b^3}{b+c}+\dfrac{c^3}{c+a} = \dfrac{3}{2} pour a=b=c=1 .

Posté par
jb2017
re : Inequation 20-07-17 à 12:16

rebonjour
@nadiasoeur123, je veux bien mais dans la disjonction des cas est-ce que tu n'aurais pas oublié un cas?  

Posté par
jb2017
re : Inequation 20-07-17 à 12:18

Oui je crois car tu considère
les cas:  tous les 3 sont + grand que 1
tous les 3 sont + petit que   1
les autres cas il y en toujours un des trois (a,b,c) égal à 1.

Par exemple  a et b <1   et c>1  n'est pas envisagé.

Posté par
nadiasoeur123
re : Inequation 20-07-17 à 12:35

Bonjour ;

tu as raison .

Posté par
Alexique
re : Inequation 20-07-17 à 12:37

en effet, 1) et 2) montrent qu'il y a une des trois variables inférieure ou égale à 1 est une des trois supérieure ou égale à 1 mais pas forcément que c'est la même variable (et la dernière peut faire n'importe quoi). Il n'y a donc pas de raison pour qu'une l'une des trois variables soit toujours égale à 1 (ce que tu as l'air de considérer comme vrai).
Ex : le triplet (\sqrt[3]{\frac32}, \sqrt[3]{\frac32},0) n'est pas traité.
Par ailleurs, l'étude de fonction, si on la détaille proprement, est loin d'être simple je pense.

Posté par
Razes
re : Inequation 20-07-17 à 12:39

Bonjour,
@ nadiasoeur123
Intéressante méthode, surtout l'introduction d'étude de fonction en dernière étape qui est infaillible, mais ceci suppose qu'on connait la solution a=b=c=1, par ailleurs il y a des cas qui ne sont pas traités, car dans les 3 premier cas tu as supposé que a,b,c ont le même comportement. Normalement on doit suivre le cheminement ci-après:

a\leqslant 1; a=1; a\geqslant 1
Normalement pour chacun de ces cas, on cherche si une solution en découle directement, sinon pour chacun des cas, envisager 3 sous cas pour b ;  b\leqslant 1; b=1; b\geqslant 1 et ainsi de suite pour c si cela est nécessaire.


Je me pose la question suivante: pourquoi ne pas prendre a comme paramètre avec chacun des cas a\leqslant 1; a=1; a\geqslant 1 et étudier la fonction paramétrée comme l'a fait nadia?

Posté par
jb2017
re : Inequation 20-07-17 à 13:26

Remarque on peut se contenter du cas a<1 et a=1. En effet si a<1, une des deux autres variable sera plus grande que 1. Donc considérer le cas a>1 sera redondant.
Je rappelle que en suivant la remarque de @glapion le problème est équivalent à montrer  
que
f(a,b,c)=a^3(a+b)+b^3(b+c)+c^3(a+c)\leq 6
Cela ne rend  pas  le problème plus facile mais on passe d'une minoration à une majoration. Du point de vue calculs tout est polynomial,...
Je continue mes remarques sur cette version équivalente .
Il y a le cas max((a+b),(b+c),(a+c)\leq 2 qui ne pose pas de problème.
En effet dans ce cas f(a,b,c) est visiblement plus petit que 6.

Posté par
DOMOREA
re : Inequation 20-07-17 à 14:02

bonjour,
Comme la piste que j'ai proposée est présente car complétée en plusieurs endroits du fil de la discussion et avec des parties inutiles ou fausses, je vous propose ici une mise en ordre en supprimant les scories.
1. je pense avoir justifié que la fonction f(a,b,c)=\frac{a^3}{a+b}+\frac{c^3}{c+a}+\frac{b^3}{b+c} est convexe sur le domaine défini par a^3+b^3+c^3=3 mais je rappelle mes arguments:
g: (x,y) \longrightarrow  \frac{x^3}{x+y}= x^3 \times \frac{1}{x+y} produit de deux fonctions convexes positives donc convexe
et f(a,b,c)=g(a,b)+g(c,a)+g(b,c) somme de 3 fonctions convexes positives donc convexe.
2. On sait que f(1,1,1)=\frac{3}{2}
en raison de la symétrie de l'expression  de  f(a,b,c), s'il existait un triplet (a_0;b_0,c_0) \neq (1,1,1) tel que f(a_0,b_0,c_0)<\frac{3}{2}    (deux au moins   des éléments de ce triplet sont distincts)  
alors on aurait ainsi  au moins 3 triplets distincts parmi les  suivants (a_0,b_0,c_0);(b_0,c_0,a_0); (c0,a_0,b_0); (c_0,b_0,a_0);  (b_0,a_0,c_0); (c_0,a_0,b_0)  dont l'image par f serait inférieure à \frac{3}{2} or ceci contredit la convexité de la surface définie sur le domaine triangulaire équilatéral D(a,b,c) défini par la relation a^3+b^3+c^3=3 le minimum est obtenu au centre de gravité  de D
est-ce convaincant ?

Posté par
Razes
re : Inequation 20-07-17 à 14:06

@jb2017
Tu as raison, les deux cas suffisent, les autres en découlent.

Posté par
DOMOREA
re : Inequation 20-07-17 à 17:54

re,
toujours pas de réaction à ma méthode ?
j'attends des objections pour m'expliquer plus avant car j'ai des arguments qui suivent

1 2 3 +




Vous devez être membre accéder à ce service...

Pas encore inscrit ?

1 compte par personne, multi-compte interdit !

Ou identifiez-vous :


Rester sur la page

Inscription gratuite

Fiches en rapport

parmi 1675 fiches de maths

Désolé, votre version d'Internet Explorer est plus que périmée ! Merci de le mettre à jour ou de télécharger Firefox ou Google Chrome pour utiliser le site. Votre ordinateur vous remerciera !